Теорема Стоуна-фон Неймана

Согласно теореме Стоуна-фон Неймана любые два канонически сопряженных самосопряженных оператора, следующих соотношению:

[ д ^ , п ^ ] "=" я
не могут быть обе ограничены. Я смущен тем, как мы доказываем эту часть и что это означает физически? Кто-нибудь может объяснить?

Нет, это не следствие теоремы Стоуна-фон Неймана о том, что два оператора (или хотя бы один) должны быть неограниченными, это следствие теоремы Виландта-Винтнера.

Ответы (3)

Я заметил, что теорема Стоуна-фон Неймана не является доказательством утверждения в начале вопроса. Оригинальные доказательства теоремы Виландта-Винтнера (кстати, доказанной только в 1947-1948 гг., в то время как теорема Стоуна-фон Неймана имела удовлетворительное доказательство фон Неймана уже к 1931 г.) находятся в:

Винтнер, А. - Неограниченность квантово-механических матриц (1947, The Physical Review, Vol. 71, p. 738-739)

Wielandt, H. - Über die Unbeschränktheit der Operatoren der Quantenmechanik (1948, Mathematische Annalen, стр. 21).

Суть доказательства Виландта в примечании 6 цитируемой страницы Вики:

введите описание изображения здесь

Значение наличия неограниченных операторов координаты и импульса на действительной оси (1D) состоит в том, что «квантовое движение» частицы не ограничено в том смысле, что либо координата, либо импульс могут быть измерены до произвольно высокого значения (бесконечного в предел), т. е. математически неограниченные операторы не имеют ограниченного спектра.

Хорошо, но что произойдет, если мы применим тот же подход к соотношению неопределенности энергии и времени, поскольку мы можем найти энергию? Мы говорим, что оператора времени не существует, поскольку гамильтониан ограничен снизу, но почему?
Это предмет одного из мифов КМ, так называемой теоремы Паули. Я вернусь к этой теме. Николич поверхностно рассматривает его здесь: arxiv.org/abs/quant-ph/0609163 .
Откровенно сэр, некоторые моменты в данной статье кажутся неверными, я думаю, что она довольно ненадежна, потому что она пытается представить все с уверенностью.
Я знаю, что это частично неверно, но Found.Phys. имеет слабый процесс рецензирования, поэтому иногда публикуется даже мусор.
Ваши дополнительные вопросы в комментариях могут быть связаны с этим вопросом: physics.stackexchange.com/q/376822 . Проверьте мой ответ там.

Теорема: Если два (не обязательно самосопряженных) ограниченных оператора д ^ и п ^ в гильбертовом пространстве удовлетворяют CCR

(1) [ д ^ , п ^ ]   "="   я   1 , я   е   С ,
затем я "=" 0 .

Косвенное доказательство: (это, по существу, доказательство работы 1.) Предположим,

(2) я     0.
С п ^ ограничено, мы можем сдвинуть
(3) п ^   "="   п ^ + б 1
на конечную положительную сумму б > 0 , так что п ^ является обратимым оператором, поэтому п ^ и п ^ 1 оба являются ограниченными операторами. Обратите внимание, что первичный оператор п ^ также удовлетворяет CCR (1). С этого момента опустим простые обозначения. Спектры _
(4) о ( д ^ п ^ )   "="   о ( п ^ д ^ п ^ п ^ 1 )   "="   о ( п ^ д ^ )
ограниченных операторов д ^ п ^ и п ^ д ^ должны быть равными ограниченными множествами. С другой стороны, CCR (1) показывает, что спектры сдвинуты
(5) о ( д ^ п ^ )   "=" ( 1 )   о ( п ^ д ^ ) + я
Единственный способ, которым экв. (2), (4) и (5) могут не противоречить друг другу, если спектры представляют собой пустые множества. Однако это противоречит общему факту (упомянутому, например, в Википедии и MO.SE ), что

Факт: каждый ограниченный оператор имеет непустой спектр.

Замечание: Если мы дополнительно предположим, что д ^ и п ^ являются самосопряженными, нам не нужно использовать указанный выше факт. Тогда коммутатор (1) антисамосопряжен, так что е р должно быть реальным. Кроме того, ограниченный оператор

(6) с ^   "="   д ^ п ^ я 2   "=" ( 1 )   п ^ д ^ + я 2   "="   с ^ ,
является самосопряженным и, следовательно, (из спектральной теоремы ) имеет непустой вещественный спектр
(7)     о ( с ^ )   "=" ( 6 )   о ( д ^ п ^ ) я 2   "=" ( 1 )   о ( п ^ д ^ ) + я 2 ,
что экв. (5) без лазейки пустых множеств.

Использованная литература:

  1. А. Винтер, Phys. 71 (1947) 738 .
Можете ли вы сказать мне, что было бы в результате применения оператора к переведенному состоянию, если бы оно действительно было ограничено?
Сэр, я студент, который только начал изучать квантовую механику, и я все еще не понимаю, как мы можем сказать, что оператор не должен быть ограниченным?
Это доказательство неверно на очень многих уровнях. Вы не можете выбрать нормализованное собственное состояние. Это не гарантируется, просто потому, что оператор является самосопряженным и ограниченным (примером является координатный оператор для частицы в конечном ящике). (5) бессмысленно, потому что операторная норма является супремумом всех q, поэтому q' не может быть больше самого себя! На самом деле (3) неверно, ибо норма (ввиду абсурдности существования) | д ) равно норме | д , потому что комплексная экспонента самосопряженного оператора является изометрическим оператором!!
Упс, теорема, о которой я думаю, кажется, работает только с дополнительным предположением, что оператор компактен. Обновленный ответ с новым доказательством.
@Qмеханик. Новое доказательство также неверно. Откуда взялось (4)? Первое равенство верно тогда и только тогда, когда p унитарно (ограниченно и изометрично), но p предполагалось самосопряженным. Единственным унитарным и самосопряженным оператором является единичный оператор. Если п ^ "=" 1 ^ , то CCR не имеет смысла.

Сопряженные переменные/операторы связаны преобразованием Фурье, то есть (квантовые) состояния одной наблюдаемой являются преобразованием Фурье другого, и поэтому только одно из них может иметь компактную поддержку (если только это не нулевая функция). Это известно как отношение неопределенности в преобразованиях Фурье . Интуитивно это означает, что разброс переменной и ее двойственное значение Фурье обратно пропорциональны, что физически выражается, например, в том, что положение локализовано (концентрировано), а импульс делокализован (разбросан). Доказательный подход см. в ответе Qmechanic.

Физически все такие типы переменных/наблюдаемых несовместимы (некоммутирующие Икс п п Икс 0 , где п Ф 1 Икс Ф с Ф : л 2 ( р ) л 2 ( р ) ), так как они не могут быть измерены одновременно с произвольной точностью. Другими словами, неопределенности двух переменных всегда ограничены средним значением их коммутатора (даже если вы производили измерения по отдельности на ансамбле из бесконечного числа одинаково подготовленных квантовых систем). Эти неопределенности являются неотъемлемым свойством любого квантового состояния.

Что такое преобразование Фурье оператора? Оператор опыт ( Икс 2 ) ограничен. Какой у него ФТ? это опыт ( п 2 ) ? Если это так, то он тоже ограничен, что противоречит вашему утверждению.
Можете ли вы объяснить это с точки зрения энергии и времени, так как они тоже канонически сопряжены, они также подчиняются принципу неопределенности, но энергию можно вычислить?
@HarshdeepSingh, это несколько особый тип соотношения неопределенностей, но он хорошо обсуждается в физике SE, см., Например,: physics.stackexchange.com/questions/53802/…
@AccidentalFourierTransform Конечно, Ф Т является унитарным оператором, и если вы сопрягаете с ним любой ограниченный оператор, он должен оставаться ограниченным. Скажем, для положения и импульса оператор положения, сопряженный с Ф , то есть п Ф 1 Икс Ф с Ф здесь л 2 нормализованный FT, дает вам оператор импульса. С другой стороны, что касается соотношения неопределенностей в КМ, опять же для положения и импульса, преобразование Фурье применяется к самим квантовым состояниям (и затем следует обсуждение компактности), неопределенность в терминах операторов выражается как CCR.
@AccidentalFourierTransform Кроме того, я немного перефразировал вещи, чтобы избежать путаницы.